Complex Analysis Inequality.












0












$begingroup$


Im studying for an exam and this is a practice problem. Not sure where to start or what theorems to even start with.



if $f$ is analytic on $|z|<1$ and $|f(z)|<1$ for $|z|<1$, prove that $$left|frac{f(z)-f(0)}{z}right|leq left|1-overline{f(0)}f(z)right|,$$



for all $0<|z|<1$.



thanks!










share|cite|improve this question









New contributor




Alex Sampson is a new contributor to this site. Take care in asking for clarification, commenting, and answering.
Check out our Code of Conduct.







$endgroup$












  • $begingroup$
    Have you seen the Schwarz-Pick theorem?
    $endgroup$
    – lEm
    Jan 8 at 5:03










  • $begingroup$
    I have not. I just looked it up and I think the proof is clear from there. Thank you.
    $endgroup$
    – Alex Sampson
    Jan 8 at 5:32
















0












$begingroup$


Im studying for an exam and this is a practice problem. Not sure where to start or what theorems to even start with.



if $f$ is analytic on $|z|<1$ and $|f(z)|<1$ for $|z|<1$, prove that $$left|frac{f(z)-f(0)}{z}right|leq left|1-overline{f(0)}f(z)right|,$$



for all $0<|z|<1$.



thanks!










share|cite|improve this question









New contributor




Alex Sampson is a new contributor to this site. Take care in asking for clarification, commenting, and answering.
Check out our Code of Conduct.







$endgroup$












  • $begingroup$
    Have you seen the Schwarz-Pick theorem?
    $endgroup$
    – lEm
    Jan 8 at 5:03










  • $begingroup$
    I have not. I just looked it up and I think the proof is clear from there. Thank you.
    $endgroup$
    – Alex Sampson
    Jan 8 at 5:32














0












0








0





$begingroup$


Im studying for an exam and this is a practice problem. Not sure where to start or what theorems to even start with.



if $f$ is analytic on $|z|<1$ and $|f(z)|<1$ for $|z|<1$, prove that $$left|frac{f(z)-f(0)}{z}right|leq left|1-overline{f(0)}f(z)right|,$$



for all $0<|z|<1$.



thanks!










share|cite|improve this question









New contributor




Alex Sampson is a new contributor to this site. Take care in asking for clarification, commenting, and answering.
Check out our Code of Conduct.







$endgroup$




Im studying for an exam and this is a practice problem. Not sure where to start or what theorems to even start with.



if $f$ is analytic on $|z|<1$ and $|f(z)|<1$ for $|z|<1$, prove that $$left|frac{f(z)-f(0)}{z}right|leq left|1-overline{f(0)}f(z)right|,$$



for all $0<|z|<1$.



thanks!







complex-analysis






share|cite|improve this question









New contributor




Alex Sampson is a new contributor to this site. Take care in asking for clarification, commenting, and answering.
Check out our Code of Conduct.











share|cite|improve this question









New contributor




Alex Sampson is a new contributor to this site. Take care in asking for clarification, commenting, and answering.
Check out our Code of Conduct.









share|cite|improve this question




share|cite|improve this question








edited Jan 8 at 6:20









Riemann

3,3281321




3,3281321






New contributor




Alex Sampson is a new contributor to this site. Take care in asking for clarification, commenting, and answering.
Check out our Code of Conduct.









asked Jan 8 at 4:50









Alex SampsonAlex Sampson

1




1




New contributor




Alex Sampson is a new contributor to this site. Take care in asking for clarification, commenting, and answering.
Check out our Code of Conduct.





New contributor





Alex Sampson is a new contributor to this site. Take care in asking for clarification, commenting, and answering.
Check out our Code of Conduct.






Alex Sampson is a new contributor to this site. Take care in asking for clarification, commenting, and answering.
Check out our Code of Conduct.












  • $begingroup$
    Have you seen the Schwarz-Pick theorem?
    $endgroup$
    – lEm
    Jan 8 at 5:03










  • $begingroup$
    I have not. I just looked it up and I think the proof is clear from there. Thank you.
    $endgroup$
    – Alex Sampson
    Jan 8 at 5:32


















  • $begingroup$
    Have you seen the Schwarz-Pick theorem?
    $endgroup$
    – lEm
    Jan 8 at 5:03










  • $begingroup$
    I have not. I just looked it up and I think the proof is clear from there. Thank you.
    $endgroup$
    – Alex Sampson
    Jan 8 at 5:32
















$begingroup$
Have you seen the Schwarz-Pick theorem?
$endgroup$
– lEm
Jan 8 at 5:03




$begingroup$
Have you seen the Schwarz-Pick theorem?
$endgroup$
– lEm
Jan 8 at 5:03












$begingroup$
I have not. I just looked it up and I think the proof is clear from there. Thank you.
$endgroup$
– Alex Sampson
Jan 8 at 5:32




$begingroup$
I have not. I just looked it up and I think the proof is clear from there. Thank you.
$endgroup$
– Alex Sampson
Jan 8 at 5:32










1 Answer
1






active

oldest

votes


















4












$begingroup$

Let $phi (z)=frac {z-f(0)} {1-overline {f(0)} z}$. Then $phi$ is holomorphic, maps the open unit disk into itself and $phi(f(0))=0$. Hence $phi circ f$ maps the open unit disk into itself, maps $0$ to $0$. By Schwarz Lemma $|phi circ f(z)| leq |z|$ which is precisely what you are trying to prove.






share|cite|improve this answer









$endgroup$













    Your Answer





    StackExchange.ifUsing("editor", function () {
    return StackExchange.using("mathjaxEditing", function () {
    StackExchange.MarkdownEditor.creationCallbacks.add(function (editor, postfix) {
    StackExchange.mathjaxEditing.prepareWmdForMathJax(editor, postfix, [["$", "$"], ["\\(","\\)"]]);
    });
    });
    }, "mathjax-editing");

    StackExchange.ready(function() {
    var channelOptions = {
    tags: "".split(" "),
    id: "69"
    };
    initTagRenderer("".split(" "), "".split(" "), channelOptions);

    StackExchange.using("externalEditor", function() {
    // Have to fire editor after snippets, if snippets enabled
    if (StackExchange.settings.snippets.snippetsEnabled) {
    StackExchange.using("snippets", function() {
    createEditor();
    });
    }
    else {
    createEditor();
    }
    });

    function createEditor() {
    StackExchange.prepareEditor({
    heartbeatType: 'answer',
    autoActivateHeartbeat: false,
    convertImagesToLinks: true,
    noModals: true,
    showLowRepImageUploadWarning: true,
    reputationToPostImages: 10,
    bindNavPrevention: true,
    postfix: "",
    imageUploader: {
    brandingHtml: "Powered by u003ca class="icon-imgur-white" href="https://imgur.com/"u003eu003c/au003e",
    contentPolicyHtml: "User contributions licensed under u003ca href="https://creativecommons.org/licenses/by-sa/3.0/"u003ecc by-sa 3.0 with attribution requiredu003c/au003e u003ca href="https://stackoverflow.com/legal/content-policy"u003e(content policy)u003c/au003e",
    allowUrls: true
    },
    noCode: true, onDemand: true,
    discardSelector: ".discard-answer"
    ,immediatelyShowMarkdownHelp:true
    });


    }
    });






    Alex Sampson is a new contributor. Be nice, and check out our Code of Conduct.










    draft saved

    draft discarded


















    StackExchange.ready(
    function () {
    StackExchange.openid.initPostLogin('.new-post-login', 'https%3a%2f%2fmath.stackexchange.com%2fquestions%2f3065811%2fcomplex-analysis-inequality%23new-answer', 'question_page');
    }
    );

    Post as a guest















    Required, but never shown

























    1 Answer
    1






    active

    oldest

    votes








    1 Answer
    1






    active

    oldest

    votes









    active

    oldest

    votes






    active

    oldest

    votes









    4












    $begingroup$

    Let $phi (z)=frac {z-f(0)} {1-overline {f(0)} z}$. Then $phi$ is holomorphic, maps the open unit disk into itself and $phi(f(0))=0$. Hence $phi circ f$ maps the open unit disk into itself, maps $0$ to $0$. By Schwarz Lemma $|phi circ f(z)| leq |z|$ which is precisely what you are trying to prove.






    share|cite|improve this answer









    $endgroup$


















      4












      $begingroup$

      Let $phi (z)=frac {z-f(0)} {1-overline {f(0)} z}$. Then $phi$ is holomorphic, maps the open unit disk into itself and $phi(f(0))=0$. Hence $phi circ f$ maps the open unit disk into itself, maps $0$ to $0$. By Schwarz Lemma $|phi circ f(z)| leq |z|$ which is precisely what you are trying to prove.






      share|cite|improve this answer









      $endgroup$
















        4












        4








        4





        $begingroup$

        Let $phi (z)=frac {z-f(0)} {1-overline {f(0)} z}$. Then $phi$ is holomorphic, maps the open unit disk into itself and $phi(f(0))=0$. Hence $phi circ f$ maps the open unit disk into itself, maps $0$ to $0$. By Schwarz Lemma $|phi circ f(z)| leq |z|$ which is precisely what you are trying to prove.






        share|cite|improve this answer









        $endgroup$



        Let $phi (z)=frac {z-f(0)} {1-overline {f(0)} z}$. Then $phi$ is holomorphic, maps the open unit disk into itself and $phi(f(0))=0$. Hence $phi circ f$ maps the open unit disk into itself, maps $0$ to $0$. By Schwarz Lemma $|phi circ f(z)| leq |z|$ which is precisely what you are trying to prove.







        share|cite|improve this answer












        share|cite|improve this answer



        share|cite|improve this answer










        answered Jan 8 at 5:33









        Kavi Rama MurthyKavi Rama Murthy

        53.2k32055




        53.2k32055






















            Alex Sampson is a new contributor. Be nice, and check out our Code of Conduct.










            draft saved

            draft discarded


















            Alex Sampson is a new contributor. Be nice, and check out our Code of Conduct.













            Alex Sampson is a new contributor. Be nice, and check out our Code of Conduct.












            Alex Sampson is a new contributor. Be nice, and check out our Code of Conduct.
















            Thanks for contributing an answer to Mathematics Stack Exchange!


            • Please be sure to answer the question. Provide details and share your research!

            But avoid



            • Asking for help, clarification, or responding to other answers.

            • Making statements based on opinion; back them up with references or personal experience.


            Use MathJax to format equations. MathJax reference.


            To learn more, see our tips on writing great answers.




            draft saved


            draft discarded














            StackExchange.ready(
            function () {
            StackExchange.openid.initPostLogin('.new-post-login', 'https%3a%2f%2fmath.stackexchange.com%2fquestions%2f3065811%2fcomplex-analysis-inequality%23new-answer', 'question_page');
            }
            );

            Post as a guest















            Required, but never shown





















































            Required, but never shown














            Required, but never shown












            Required, but never shown







            Required, but never shown

































            Required, but never shown














            Required, but never shown












            Required, but never shown







            Required, but never shown







            Popular posts from this blog

            Mario Kart Wii

            What does “Dominus providebit” mean?

            Antonio Litta Visconti Arese